Merge remote branch 'public/master'
[course.git] / latex / problems / Serway_and_Jewett_4 / problem08.06.tex
1 \begin{problem*}{8.6}
2 A friend claims that as long as he has his seat belt on, he can hold
3 on to an $m = 12.0\U{kg}$ child in a $v_i = 60.0\U{mph}$ head-on
4 collision with a brick wall in which the car passenger compartment
5 comes to a stop in $\Delta t = 0.050\U{s}$.  Show that the violent
6 force during the collision will tear the child from his arms.
7 \end{problem*}
8
9 \begin{solution}
10 The force needed to hold on to the child is given by
11 \begin{equation}
12  F = \frac{\Delta p}{\Delta t} = - m \frac{v_i}{\Delta t}
13    = -12.0\U{kg}\frac{60\U{mph}}{0.050\U{s}}
14      \cdot \frac{1609\U{m}}{1\U{mi}} \cdot \frac{1\U{hr}}{3600\U{s}}
15    = -6436\U{N}
16 \end{equation}
17 Which is much larger than what the friend is capable of applying.
18 \end{solution}